MENTAL HEALTH (PSYCH) HESI – FINAL EXAM PRACTICE QUESTIONS & ANSWERS

The nurse is discussing the grieving process with the client. Which stages are included in Kubler-Ross’ stages of grief? Rank in the correct order.

  1. Acceptance
  2. Bargaining
  3. Denial
  4. Anger
  5. Depression
    Answer:
  6. Denial
  7. Anger
  8. Bargaining
  9. Depression
  10. Acceptance

Which situation requires priority intervention on an inpatient psychiatric unit?

  1. A client is threatening to throw the television at another client.
  2. A male client wants to use the phone to call his spouse.
  3. A client sitting in a chair is delusional and hallucinating
  4. A client has refused to eat anything for the last 2 days.
    Answer:
  5. A client is threatening to throw the television at another client.

The client diagnosed with bipolar disorder and who is prescribed lithium, an anti mania medication, is admitted to the psychiatric unit in an acute manic state. Which intervention should the nurse implement first?

  1. Have the laboratory draw a STAT serum lithium level.
  2. Evaluate what behavior prompted the psychiatric admission.
  3. Assess and treat the client’s physiological needs.
  4. Administer STAT dose of lithium to the client.
    Answer:
  5. Assess and treat the client’s physiological needs.

The psychiatric unit staff is upset about the new female charge nurse who just sits in her office all day. One of the staff members informs the clinical manager indicates a laissez-faire leadership style?

  1. “I will schedule a meeting to discuss the concerns of the charge nurse.”
  2. “I hired the new charge nurse and she is doing what I told her to do.”
  3. “You and the staff really should take care of this situation on your own.”
  4. “I will talk to the charge nurse about your concerns and get back to you.”
    Answer:
  5. “You and the staff really should take care of this situation on your own.”
    (laissez-faire attitude is an “everybody is driving their own train” attitude, you guys take care of it)(option 1. is democratic. 2. is an autocratic style. authoritarian approach. 4. is taking charge of the situation.)

The mental health worker reports that one of the nurses threatened to force-fed the male client diagnosed with schizophrenia if the client did not eat the meal on the lunch tray. Which action should the charge nurse take first?

  1. Tell the MHW that this intervention is part of the client’s care plan.
  2. Request the nurse to come to the office and discuss the MHW allegation.
  3. Ask the client what happened between him and the nurse during lunch.
  4. Ask the MHW to write down the situation to submit to the head nurse.
    Answer:
  5. Request the nurse to come to the office and discuss the MHW allegation.
    (never choose an answer like 4. where it suggests you “pass it up” so that you don’t have to deal with the problem. it will be wrong)

The client diagnosed with paranoid schizophrenia is imminently aggressive and is dangerous to himself, the other clients, and the psychiatric staff members. The client is placed in a seclusion room. Which interventions should the psychiatric nurse implement? Select all that apply.

  1. Assess the client every 2 hours for side effects of medication.
  2. Tell the client what behavior will prompt the release from seclusion.
  3. Do not notify the client’s family of the initiation of seclusion.
  4. Explain that the client will be in seclusion room for 24 hours.
  5. Instruct the MHW to check the client every 10-15 minutes.
    Answer:
  6. Assess the client every 2 hours for side effects of the medication., 2. Tell the client what behavior will prompt the release from seclusion., 5. Instruct the MHW to check the client every 10-15 minutes.

The psychiatric nurse overhears a MHW arguing with a client diagnosed with paranoid schizophrenia. Which action should the nurse implement?

  1. Ask the MHW to go to the nurse’s station.
  2. Tell the MHW to quit arguing with the client.
  3. Notify the clinical manager of the psychiatric unit.
  4. Report the behavior to the client abuse committee.
    Answer:
  5. Ask the MHW to go to the nurse’s station. (first thing you are going to do is assess situation with the MHW.)

The psychiatric nurse has taken 15 minutes extra for the lunch break two times in the last week. Which action should the female clinical manager implement?

  1. Take no action and continue to watch the nurse’s behavior.
  2. Document the behavior in writing and place in the nurse’s file.
  3. Tell the nurse to check in and out with her when taking lunch.
  4. Talk to the nurse informally about taking 45 minutes for lunch.
    Answer:
  5. Talk to the nurse informally about taking 45 minutes for lunch. (Assess)

The client diagnosed with Alzheimer’s disease is on a special unit for clients with cognitive disorders. Which assessment data would warrant immediate intervention by the psychiatric nurse?

  1. The client does not know his or her name, date, or place.
  2. The client is unable to dress himself or herself without assistance.
  3. The client is difficult to arouse from sleep.
  4. The client needs assistance when eating a meal.
    Answer:
  5. The client is difficult to arouse from sleep.

The mother of a client recently diagnosed with schizophrenia says to the nurse, “I was afraid of my son. Will he be alright?” Which response by the psychiatric nurse supports the ethical principal of veracity?

  1. “I can see your fear; you are concerned your son will not be alright.”
  2. “If your son takes medication, the symptoms can be controlled.”
  3. “Why were you afraid of your son? Did you think he would hurt you?”
  4. “Schizophrenia is a mental ill ness and your son will not be alright.”
    Answer:
  5. “If your son takes medication, the symptoms can be controlled.”
    (veracity means to tell the truth. 1. is a therapeutic response. 3. don’t ask the pt why or what made you. 4. opposite of veracity, that is not true.

The client in the psychiatric unit tells a nurse, “Someone just put a bomb under the couch in the lobby.” Which action should the nurse implement first?

  1. Look under the couch for a bomb.
  2. Implement the bomb scare protocol.
  3. Have the staff evacuate the unit.
  4. Tell the client there is not a bomb.
    Answer:
  5. Look under the couch for a bomb. (Assess)

The head nurse in a psychiatric unit in the county emergency department is assigning client to the staff nurses. Which client should be assigned to the most experienced nurse?

  1. The client who is crying and upset because she was raped.
  2. The client diagnosed with bipolar disorder who is agitated.
  3. The client who was found wandering the streets in a daze.
  4. The client diagnosed with schizophrenia who is hallucinating.
    Answer:
  5. The client who was found wandering the streets in a daze. (when the Q asks “who are you going to give to the most experienced nurse” what they are really asking is “who is the most unstable.” “who requires the most in-depth assessment and teaching?” we know what is wrong with all of these patients, besides #3., we only know the client was found walking in a daze, we don’t know anything about them)

the client diagnosed with anorexia is refusing to eat and is less than 20% of ideal body weight for her height and structure. The client has not eaten anything since admission 2 days ago. Which action should the nurse implement?

  1. Notify the psychiatrist to request a court order to feed the client.
  2. Take no action because the client has the right to refuse treatment.
  3. Discharge the client because she is not complying with the treatment.
  4. Physically restrain the client and insert a nasogastric tube for feeding.
    Answer:
  5. Notify the psychiatrist to request a court order to feed the client.

The nurse answers the client’s phone in the lobby area and the person asks, “May I speak to Mr. Jones?” Which action should the nurse implement?

  1. Ask the caller who is asking for Mr. Jones.
  2. Tell the caller Mr. Jones cannot have phone calls.
  3. Request the caller to give the access code for information.
  4. Find Mr. Jones and tell them he has a phone call.
    Answer:
  5. Find Mr. Jones and tell them he has a phone call. (This is the client’s phone. Facility phone would be different.)

The client being the psychiatric nurse in the mental health clinic tells the nurse “If I tell you something very important, will you promise not to tell anyone?” Which statement is the nurse’s best response?

  1. “I promise I will not tell anyone if you don’t want me to.”
  2. “If it affects your care I will have to tell someone who can help.”
  3. “If you don’t want me to tell anyone, then please don’t tell me.”
  4. “Why do you not want me to tell anyone if it is so important?”
    Answer:
  5. “If it affects your care I will have to tell someone who can help.”
    (1. don’t promise anything. 3. do not shut a pt down when they are trying to communicate. 4. never ask a pt “why” or “what made you”.)

Which situation would warrant immediate intervention by the charge nurse on the psychiatric unit after receiving the a.m. shift report?

  1. The client diagnosed with paranoid schizophrenia who is delusional.
  2. The p.m. shift licensed practical nurse called in to say that he or she would not be in for work today.
  3. The male mental health worker reports losing his unit key and identification card.
  4. The unit secretary has HCP’s orders that need to be co-signed.
    Answer:
  5. The male mental health worker reports losing his unit key and identification card. (The unit is no longer secure. Pt’s safety is at risk.)

A highly agitated client paces the unit and states. “I could buy and sell this place.” The client’s mood fluctuates rom fits of laughter to outbursts of anger. Which is the most accurate documentation of this client’s behavior?
A. “Rates mood 8/10. Exhibiting looseness of association. Euphoric.”
B. “Mood euthymic. Exhibiting magical thinking. Restless.”
C. “Mood labile. Exhibiting delusions of reference. Hyperactive.”
D. “Agitated and pacing. Exhibiting grandiosity. Mood labile.”
Answer:
D. “Agitated and pacing. Exhibiting grandiosity. Mood labile.”

A client diagnosed with bipolar disorder is distraught over insomnia experienced over the last 3 nights and a 12- pound weight loss over the past 2 weeks. Which should be this client’s priority nursing diagnosis?
A. Knowledge deficit R/T bipolar disorder AEB concern about symptoms.
B. Altered nutrition: Less than body requirements r/t hyperactivity AEB weight loss.
C. Risk for suicide r/t powerlessness AEB insomnia and anorexia.
D. Altered sleep patterns r/t mania AEB insomnia for the past 3 nights.
Answer:
B. Altered nutrition: Less than body requirements r/t hyperactivity AEB weight loss. (physical needs/health always priority.)

A nurse is planning care for a client diagnosed with bipolar disorder: manic phase. In which order should the nurse prioritize the client outcomes in the exhibit?
Client outcomes:

  1. Maintains nutritional status.
  2. Interacts appropriately with peers.
  3. Remains free from injury.
  4. Sleeps 6-8 hrs at night.
    Answer:
    3., 1., 4., 2.

A client diagnosed with bipolar disorder: depressive phase intentionally overdoses on sertraline (Zoloft). Family members report that the client has experience anorexia, insomnia, and recent job loss. What should be the priority nursing diagnosis for this client?
A. Risk for suicide r/t hopelessness.
B. Anxiety: severe r/t hyperactivity
C. Imbalanced nutrition: less than body requirements r/t refusal to eat
D. Dysfunctional grieving r/t loss of employment.
Answer:
A. Risk for suicide r/t hopelessness.

A client diagnosed with BP disorder: manic episode refuses to take lithium carbonate due to excessive weight gain. in order to increase compliance, which medication should a nurse anticipate that a physician will prescribe?
A. Sertraline (Zoloft)
B. Valporic acid (Depakote)
C. Trazodone (Desyrel)
D. Paroxetine (Paxil)
Answer:
B. Valporic acid (Depakote)
(prescribed to help with weight loss.)

A client diagnosed with BP disorder is exhibiting severe manic behaviors. A physician prescribes lithium carbonate (Eskalith) and olanzapine (Zyprexa). The client’s spouse questions the Zyprexa order. Which is the appropriate nursing response?
A. “Zyprexa in combination with Eskalith cures manic symptoms.”
B. “Zyprexa prevents extrapyramidal side effects.”
C. “Zyprexa ensures a good night’s sleep.”
D. “Zyprexa calms hyperactivity until the Eskalith takes effect.”
Answer:
D. “Zyprexa calms hyperactivity until the Eskalith takes effect.”
(when it comes to psych meds, it takes 4-6 wks to see effect. Give Zyprexa as a bridge until Lithium takes in effect.”

A client began taking lithium for the treatment of BP disorder approximately 1 month ago. The client asks if it is normal to have gained 12 pounds in this time frame. Which is the appropriate nursing response?
A. “Thats strange. Weight loss is the typical pattern.”
B. “What have you been eating? Weight gain is not usually associate with lithium.”
C. “Weight gain is a common but troubling side effect.”
D. “Weight gain only occurs during the first month of treatment with this drug.”
Answer:
C. “Weight gain is a common but troubling side effect.”

A client diagnosed with BP disorder has been taking lithium carbonate (Lithaine) for 1 year. The client presents in an emergency department with a temp of 101F (38C), severe diarrhea, blurred vision, and tinnitus. How should the nurse interpret these symptoms?
A. Symptoms indicate consumption of foods high in tyramine.
B. Symptoms indicate lithium carbonate discontinuation syndrome.
C. Symptoms indicate the development of lithium carbonate tolerance.
D. Symptoms indicate lithium carbonate toxicity.
Answer:
D. Symptoms indicate lithium carbonate toxicity. (0.6-1.2 normal range. blurred vision and tinnitus is most obvious clue)

A nursing instructor is discussing various challenges in the treatment of clients diagnosed with bp disorder. Which student statement demonstrates an understanding of the most critical challenge in the care of these clients?
A. “Treatment is compromised when clients can’t sleep.”
B. “Treatment is compromised with irritability interferes with social interactions.”
C. “Treatment is compromised when clients have no insight into their problems.”
D. “Treatment is compromised when clients choose not to take their medications.”
Answer:
D. “Treatment is compromised when clients choose not to take their medications.”

A clients diagnosed with bp disorder: manic phase. Which nursing intervention would be implemented to achieve the outcome of “Client will gain 2 lb by the end of the week?”
A. Provide client with high-calorie finger foods throughout the day.
B. Accompany client to cafeteria to encourage adequate dietary consumption.
C. Initiate total parenteral nutrition to meet dietary needs.
D. Teach the importance of a varied diet to meet nutritional needs.
Answer:
A. Provide client with high-calorie finger foods throughout the day.
(Pt is in manic phase. Finger foods will be best for them because pt cannot sit down and eat a meal, they will not slow down or stop long enough to eat a meal.)

A nurse discovers a client’s suicide note that details the time, place, and means to commit suicide. What should be the priority nursing action and why?
A. Administering lorazepam (Ativan) prn, because the client is angry at exposure of plan.
B. Establishing room restrictions, because the client’s threat is an attempt to manipulate the staff.
C. Placing this client on one-to-one suicide precautions, because the more specific the plan, the more likely the client will attempt suicide.
D. Calling an emergency treatment team meeting, because the client’s threat must be addressed.
Answer:
C. Placing this client on one-to-one suicide precautions, because the more specific the plan, the more likely the client will attempt suicide.
(First question: are you suicidal? Second: Do you have a plan? third: What is your plan? Important to assess the details of the plan.

Immediately after electroconvulsive therapy, in which position should a nurse place the client?
A. On his or her side to prevent aspiration.
B. In high Fowler’s position to promote consciousness.
C. In Trendlenburg’s posiiton to promote blood flow to vital organs.
D. In prone position to prevent airway blockage.
Answer:
A. On his or her side to prevent aspiration.

A client is diagnosed with major depressive disorder. Which nursing diagnosis should a nurse assign to this client to address a behavioral symptom of this disorder?
A. Altered communication r/t feelings of worthlessness AEB anhedonia.
B. Social isolation r/t poor self-esteem AEB secluding self in room.
C. Altered thought processes r/t hopelessness AEB persecutory delusion.
D. Altered nutrition: less than body requirements r/t high anxiety AEB anorexia.
Answer:
B. Social isolation r/t poor self-esteem AEB secluding self in room.
(KEY: question asked for BEHAVIORAL symptom.)

A client diagnosed with major depression with psychotic features hears voices commanding self-harm. A nurse is unable to elicit a contract for safety. What should be the nurse’s priority intervention at this time?
A. Obtaining an order for locked seclusion until client is no longer suicidal.
B. conducting 15-minute checks to ensure safety.
C. Placing the client on one-to-one observation while continuing to monitor suicidal ideations.
D. Encouraging client to express feelings related to suicide.
Answer:
C. Placing the client on one-to-one observation while continuing to monitor suicidal ideations.
(Command hallucinations are the first clue, pt is refusing to sign a safety contract, that pt needs to be on one-on-one and continue to monitor)

The nurse assesses a client suspected of having major depressive disorder. Which client symptom would eliminate this diagnosis?
A. The client is disheveled and malodorous.
B. The client refuses to interact with others.
C. The client is unable to feel any pleasure.
D. The client has maxed-out charge cards and exhibits promiscuous behaviors.
Answer:
D. The client has maxed-out charge cards and exhibits promiscuous behaviors.

A client with a history of suicide attempts has been taking fluoxetine (Prozac) for 1 month. The client suddenly presents with a bright affect, rates mood at 9/10, and is much more communicative. Which action should be the nurse’s priority at this time?
A. give the client off-unit privileges as positive reinforcement.
B. Encourage the client to share mood improvement in group.
C. Increase the level of this client’s suicide precautions.
D. Request that the psychiatrist reevaluate the current medication protocol.
Answer:
C. Increase the level of this client’s suicide precautions.
(No miraculous healing occurs, in 4 wks pt will not recieve effects from medication. They may have come up with a fool proof suicide plan. Pay attention to time frame that question gives you such as “1 month”)

What is the rationale for a nurse to perform a full physical health assessment on a client admitted with a diagnosis of major depressive disorder?
A. The attention during the assessment is beneficial in decreasing social isolation.
B. Depression can generate somatic symptoms that can mask actual physical disorders.
C. Physical health complications are likely to arise form antidepressant therapy.
D. Depressed clients avoid addressing physical health and ignore medical problems.
Answer:
B. Depression can generate somatic symptoms that can mask actual physical disorders.
(Concept of this question is a popular mental health question.)

A nurse is planning care for a child who is experiencing depression. Which medication is approved by the FDA for the treatment of depression in children and adolescents?
A. Paroxetine (Paxil)
B. Sertraline (Zoloft)
C. Citalopram (Celexa)
D. Fluoxetine (Prozac)
Answer:
D. Fluoxetine (Prozac)
(Famous HESI, NCLEX question. SSRI is first line treatment for depression)

A nurse recently admitted a client to an inpatient unit after a suicide attempt. A health-care provider orders amitriptyline (Elavil) for the client. Which intervention, related to this medication, should be initiated to maintain this client’s safety upon discharge?
A. Provide a 6-month supply of Elavil to ensure long-term compliance.
B. Provide a 1-wk supply of Elavil with refills contingent on follow-up appointments.
C. Provide pill dispenser as a memory aid.
D. Provide education regarding the avoidance of foods containing tyramine.
Answer:
B. Provide a 1-wk supply of Elavil with refills contingent on follow-up appointments.
(followup is good for assessment, don’t want to give enough supply for them to take all at once and overdose.)

A client who has been taking fluvoxamine (Luvox) without improvement asks a nurse “I heard about something called a monoamine oxidase inhibitor (MAOI). Cant my doctor add that to my medications?” Which is an appropriate nursing response?
A. “This combination of drugs can lead to delirium tremens.”
B. “A combination of an MAOI and Luvox can lead to a life-threatening hypertensive crisis.”
C. “Thats a good idea. There have been good results with the combination of these two drugs.”
D. “The only disadvantage would be the exorbitant cost of the MAOI.”
Answer:
B. “A combination of an MAOI and Luvox can lead to a life-threatening hypertensive crisis.”
(MAOI have many interactions. Prescribers avoid prescribing MAOIs)

A psychiatrist prescribes a monoamine oxidase inhibitor for a client. Which foods should the nurse teach the client to avoid?
A. Pepperoni pizza and red wine.
B. Bagels with cream cheese and tea.
C. Apple pie and coffee.
D. Potato chips and diet cola.
Answer:
A. Pepperoni pizza and red wine.
(Contain tyramine. No cured meats like pepperoni, no aged cheeses, no red wine, no beer.)

A client is prescribed phenelzine (Nardil). Which statements by the client should indicate to a nurse that discharge teaching about this medication has been successful? (select all that apply)
A. “Ill have to let my surgeon know about this medication before I have my cholecystectomy.”
B. “Guess I will have to give up my glass of red wine with dinner.”
C. “Ill have to be very careful about reading food and medication labels.”
D. “Im going to miss my caffeinated coffee in the morning.”
E. “Ill be sure not to stop this medication abruptly.”
Answer:
A., B., C., E.

During an assessment interview, a client diagnosed with antisocial personality disorder spits, curses, and refuses to answer questions. Which is the appropriate nursing response to this behavior?
A. “You are very disrespectful. You need to learn to control yourself.”
B. “I understand that you are angry, but this behavior will not be tolerated.”
C. “What behaviors could you modify to improve this situation?”
D. “What anti-personality disorder medications have helped you in the past?”
Answer:
B. “I understand that you are angry, but this behavior will not be tolerated.”

A client diagnosed with antisocial personality disorder comes to a nurses’ station at 11:00 p.m. requesting to phone a lawyer to discuss filing for divorce. The unit rules state that no phone calls are permitted after 10:00 p.m. Which nursing response is most appropriate?
A. “Go ahead and use the phone. I know this pending divorce is stressful.”
B. “You know better than break the rules. Im surprised at you. “
C. “It is after the 10:00 p.m. phone curfew. You will be able to call tomorrow.”
D. “The decision to divorce should not be considered until you have had a good nights sleep.”
Answer:
C. “It is after the 10:00 p.m. phone curfew. You will be able to call tomorrow.”

A client diagnosed with paranoid personality disorder becomes violent on a unit. Which nursing intervention is most appropriate?
A. Provide objective evidence that reasons for violence are unwarranted.
B. Initially restrain the client to maintain safety.
C. Use clear, calm, statements and a confident physical stance.
D. Empathize with the client’s paranoid perceptions.
Answer:
C. Use clear, calm, statements and a confident physical stance.
(First thing you are going to do when a pt becomes violent, you are going to firmly and calmly let the pt know that they need to calm down. If necessary, restrain pt.)

A highly emotional client presents at an outpatient clinic appointment wearing flamboyant attire, spiked heels, and theatrical makeup. Which personality disorder should a nurse associate with this behavior?
A. Compulsive personality disorder
B. Schizotypal personality disorder
C. Histrionic personality disorder
D. Manic personality disorder
Answer:
C. Histrionic personality disorder
(Histrionic are the drama queens, attention seeker. Underlying issue with compulsive disorder is anxiety. Schizotypal disorder, baseline is psychosis.
Manic disorder are pts who are on level 100 all of the time.)

A client diagnosed with borderline personality disorder brings up a conflict with the staff in a community meeting and develops a following of clients who unreasonably demand modification of unit rules. How can the nursing staff best handle this situation?
A. Allow the clients to apply the democratic process when developing unit rules.
B. Maintain consistency of care by open communication to avoid staff manipulation.
C. Allow the client spokesman to verbalize concerns during a unit staff meeting.
D. Maintain unit order by the application of autocratic leadership.
Answer:
B. Maintain consistency of care by open communication to avoid staff manipulation.
(borderline personality disorder is very manipulative. They want to cause splitting which looks like pitting the nurses against each other, or one day you are the best nurse ever and then the next day you are the worst nurse in the world.)

Which nursing approach should be utilized to maintain a therapeutic relationship with a client diagnosed with borderline personality disorder?
A. Being firm, consistent, and empathetic while addressing specific client behaviors.
B. Promoting client self-expression by implementing laissez-faire leadership.
C. Using authoritative leadership to help clients learn to conform to society norms.
D. Overlooking inappropriate behaviors to avoid providing secondary gains.
Answer:
A. Being firm, consistent, and empathetic while addressing specific client behaviors.

Which adult client should a nurse identify as exhibiting the characteristics of a dependent personality disorder?
A. A physically healthy client who is dependent on meeting social needs by contact with 15 cats.
B. A physically healthy client who has a history of depending on intense relationships to meet basic needs.
C. A physically healthy client who lives with parents and depends on public transportation.
D. A physically healthy client who is serious, inflexible, perfectionistic, and depends on rules to provide security.
Answer:
C. A physically healthy client who lives with parents and depends on public transportation.
(dependent personality disorder is depending on someone else to take care of them, submissive attitude. Refuses to take responsibility for anything.)

During an interview, which client statement to a nurse should indicate a potential diagnosis of schizotypal personality disorder?
A. “I really don’t have a problem. My family is inflexible, and every relative is out to get me.”
B. “I am so excited about working with you. Have you noticed my new nail polish: ‘Ruby Red Roses’?”
C. “I spend all my time tending my bees. I know a whole lot of information about bees.”
D. “I am getting a message from the beyond that we have been involved in a previous life.”
Answer:
D. “I am getting a message from the beyond that we have been involved in a previous life.”
(Schizotypal disorder is a psychosis disorder and they are out of touch with reality. Displays false beliefs and delusions.”

A nursing instructor is teaching students about clients diagnosed with histrionic personality disorder and the quality of their relationships. Which student statement indicates that learning has occurred?
A. Their dramatic style tends to make their interpersonal relationships quite interesting and fulfilling.
B. Their interpersonal relationships tend to be shallow and fleeting, serving their dependency needs.
C. They tend to develop few relationships because they are strongly independent but generally maintain deep affection.
D. They pay particular attention to details, which can frustrate the development of relationships.
Answer:
B. Their interpersonal relationships tend to be shallow and fleeting, serving their dependency needs.
(Histrionic disorder: attention seekers, will do anything for attention.)

When planning care for a patient diagnosed with borderline personality disorder, which self-harm behavior should a nurse expect the client to exhibit?
A. The use of highly lethal methods to commit suicide.
B. The use of suicidal gestures to elicit a rescue response from others.
C. The use of isolation and starvation as suicidal methods.
D. The use of self-mutilation to decreased endorphins in the body.
Answer:
B. The use of suicidal gestures to elicit a rescue response from others.

Which nursing diagnosis should be prioritized when providing nursing care to a client diagnosed with paranoid personality disorder?
A. Risk for violence: directed toward others r/t paranoid thinking.
B. Risk for suicide r/t altered thought
C. Altered sensory perception r/t increased levels of anxiety
D. Suicidal isolation r/t inability to relate to others.
Answer:
A. risk for violence: directed toward others r/t paranoid thinking.

A paranoid client presents with bizarre behaviors, neologisms, and thought insertion. Which nursing action should be prioritized to maintain this client’s safety?
A. Assess for medication noncompliance.
B. Note escalating behaviors and intervene immediately.
C. Interpret attempts at communication.
D. Assess triggers for bizarre, inappropriate behaviors.
Answer:
B. Note escalating behaviors and intervene immediately.
(ex escalating behaviors: pacing, hand tapping or fidgeting, facial expression. Other options are not as big of a priority.)

A client diagnosed with schizoaffective disorder is admitted for social skills training. Which information should be included in the nurse’s teaching?
A. The side effects of medications.
B. Deep breathing techniques to decrease stress
C. How to make eye contact when communicating
D. How to be a leader.
Answer:
C. How to make eye contact when communicating
(Look at the question, it asks for social skills training. All are good answers, test writers try and trick you.)

A client diagnosed with schizophrenia tells a nurse, “The Shopatouliens took my shoes out of my room last night”. Which is an appropriate charting entry to describe this client’s statement?
A. “The client is experiencing command hallucinations”
B. “The client is expressing a neologism.”
C. “The client is experiencing a paranoid delusion.”
D. “The client is verbalizing a word salad.”
Answer:
B. “The client is expressing a neologism.”
(neologism means words that don’t make sense, or pt is making words up. Pt is not experiencing command hallucinations – the most dangerous kind of hallucinations. A paranoid delusion is having a belief that is not true or real. Word salad is using a bunch of words in a sentence that don’t make sense.)

A client diagnosed with schizophrenia states, “Cant you hear him? It’s the devil. He’s telling me I’m going to hell.” Which is the most appropriate nursing response?
A. Did you take your medicine this morning?
B. You are not going to hell. You are a good person.
C. I am sure the voices sound scary, but the devil is not talking to you. This is part of your illness.
D. The devil only talks to people who are receptive to his influence.
Answer:
C. I am sure the voices sound scary, but the devil is not talking to you. This is part of your illness.
(Most therapeutic response. Acknowledge their feelings, and also bringing them back to reality. Avoid using yes or no answered questions except when asking important questions such as “do you have any allergies”)

Which nursing intervention would be most appropriate when caring for an acutely agitated client diagnosed with paranoid schizophrenia?
A. Provide neon lights and soft music.
B. Maintain continual eye contact throughout the interview.
C. Use therapeutic touch to increase trust and rapport.
D. Provide personal space to respect the client’s boundaries.
Answer:
D. Provide personal space to respect the client’s boundaries.
(Look at the question, “acutely agitated”. agitated right now.)

Which nursing behavior will enhance the establishment of a trusting relationship with a client diagnosed with schizophrenia?
A. Establishing personal contact with family members.
B. Being reliable, honest, and consistent during interactions.
C. Sharing limited personal information.
D. Sitting close to the client to establish rapport.
Answer:
B. Being reliable, honest, and consistent during interactions.
(Pts with schizophrenia are paranoid, important to be consistent and build trust.)

A client diagnosed with paranoid schizophrenia states, “My psychiatrist is out to get me. Im sad that the voice is telling me to stop him.” What symptom is the client exhibiting and what is the nurses legal responsibility related to this symptom?
A. Magical thinking; administer an antipsychotic medication.
B. Persecutory delusions; orient the client to reality.
C. Command hallucinations; warn the psychiatrist.
D. Altered thought processes; call an emergency treatment team meeting.
Answer:
C. Command hallucinations; warn the psychiatrist.

Which statement should indicate to a nurse that an individual is experiencing a delusion?
A. Theres an alien growing in my liver.
B. I see my dead husband everywhere I go.
C. The IRS may audit my taxes.
D. Im not going to eat my food. It smells like brimstone.
Answer:
A. Theres an alien growing in my liver.
(A delusion is a false belief. Seeing something or hearing something that is not real is a hallucination)

A client diagnosed with schizophrenia is slow to respond and appears to be listening to unseen others. Which medication should a nurse expect a physician to order to address this type of symptom?
A. Haloperidol (Haldol) to address the negative symptom.
B. Clonazepam (Klonopin) to address the positive symptom.
C. Risperidone (Risperdal) to address the positive symptom.
D. Clozapine (Clozaril) to address the negative symptom.
Answer:
C. Risperidone (Risperdal) to address the positive symptom.
(Risperdal is a antipsychotic that would be given for a pt who is in psychosis, this pt is exhibiting positive symptoms. Positive symptoms are symptoms that the pt has that they should not have such as seeing things that are not there. Negative symptoms are the pt not having things that they should be having such as facial expression.)

A client is diagnosed with schizophrenia. A physician orders haloperidol (Haldol) 50 mg bid, benztropine (Cogentin) 1 mg pro, and zolpidem (Ambien) 10 mg HS. Which client behavior would warrant the nurse to administer benztropine?
A. Tactile hallucinations
B. Tardive dyskinesia
C. Restlessness and muscle rigidity
D. Reports of hearing disturbing voices
Answer:
C. Restlessness and muscle rigidity
(Tardive dyskinesia would be because antipsychotic medications)

A nurse is caring for a client who is experiencing a flat affect, paranoid delusions, anhedonia, anergia, neologisms, and echolalia. Which statement correctly differentiates the client’s positive and negative symptoms of schizophrenia?
A. Paranoid delusions, anhedonia, and anergia are positive symptoms of schizophrenia.
B. Paranoid delusions, neologisms, and echolalia are positive symptoms of schizophrenia.
C. Paranoid delusions, anergia, and echolalia are negative symptoms of schizophrenia .
D. Paranoid delusions, flat affect, and anhedonia are negative symptoms of schizophrenia.
Answer:
B. Paranoid delusions, neologisms, and echolalia are positive symptoms of schizophrenia.

A 60 year old client diagnosed with chronic schizophrenia presents in an emergency department with uncontrollable tongue movements, stiff neck, and difficulty swallowing. Which medical diagnosis and treatment should a nurse anticipate when planning care for this client?
A. Neuroleptic malignant syndrome treated by discontinuing antipsychotic medications.
B. Agranulocytosis treated by administration of clozapine (Clozaril).
C. Extrapyramidal symptoms treated by administration of benztropine (Cogentin).
D. Tardive dyskinesia treated by discontinuing antipsychotic medications.
Answer:
D. Tardive dyskinesia treated by discontinuing antipsychotic medications.
(happens after pt has chronically taken antipsychotic medications. When prescribing antipsychotics, you want to start low and gradually increase dose because of this. Question clues: “Chronic schizophrenia”, “uncontrollable tongue movements, stiff neck, difficulty swallowing”)

After taking chlorpromazine (Thorazine) for 1 month, a client presents to an emergency department with severe muscle rigidity, tachycardia, and a temp of 105F (40.5C). Which medical diagnosis and treatment should a nurse anticipate when planning care for this client?
A. Neuroleptic malignant syndrome treated by discontinuing Thorazine and administering dantrolene (Dantrium).
B. Neuroleptic malignant syndrome treated by increasing thorazine dosage and administering an anti anxiety medication.
C. Distonia treated by administering trihexyphenidyl (Artane).
D. Dystonia treated by administering bromocriptine (Parlodel).
Answer:
A. Neuroleptic malignant syndrome treated by discontinuing Thorazine and administering dantrolene (Dantrium).
(dantrolene (Dantrium) is a muscle relaxant).

A client diagnosed with schizophrenia takes an antipsychotic agent daily. Which assessment finding should a nurse address first?
A. Respirations of 22 beats/minute
B. Weight gain of 8 pounds in 2 months
C. Temperature of 104 F (40C)
D. Excessive salivation
Answer:
C. Temperature of 104 F (40C)
(Concerned about a seizure. More concerning if respirations are decreased.)

An aging client diagnosed with chronic schizophrenia takes an antipsychotic and a beta-adrenergic blocking agent (propranolol) for hypertension. Understanding the combined side effects of these drugs, which statement by a nurse is most appropriate?
A. Make sure you concentrate on taking slow, deep, cleansing breaths.
B. Watch your diet and try to engage in some regular physical activity.
C. Rise slowly when you change position from lying to sitting or sitting to standing.
D. Wear sunscreen and try to avoid midday sun exposure.
Answer:
C. Rise slowly when you change position from lying to sitting or sitting to standing.

If clozapine (Clozaril) therapy is being considered, which laboratory test should a nurse review to establish a baseline for comparison to evaluate a potentially life-threatening side effect?
A. White blood cell count
B. Liver function studies
C. Creatinine clearance
D. Blood urea nitrogen
Answer:
A. White blood cell count
(Adverse effect is agranulocytosis)

Which components should a nurse recognize as an integral part of a rehabilitative program when planning care for clients diagnosed with schizophrenia? (select all that apply)
A. Group therapy
B. Medication management
C. Deterrent therapy
D. Supportive family therapy
E. Social skills training
Answer:
A., B., D., E.

A nurse administering risperidone (Risperdal) to a client diagnosed with schizophrenia. Which symptoms should a nurse expect the therapeutic effect of this medication to address? (select all that apply)
A. Somatic delusions
B. Social isolation
C. Gustatory hallucinations
D. Flat affect
E. Clang associations
Answer:
A., C., E.

Family dynamics are thought to be a major influence in the development of anorexia nervosa. Which statement regarding a client’s home environment should a nurse associate with the development of anorexia nervosa?
A. The home environment maintains loose personal boundaries.
B. The home environment places an overemphasis on food.
C. The home environment is overprotective and demands perfection
D. The home environment condones corporal punishment.
Answer:
C. The home environment is overprotective and demands perfection
(Pt with anorexia nervosa tends to be overachievers. Everything around them is controlled, the only thing they can control is their body and what they put into their body.)

A client’s altered body image is evidence by claims of “feeling fat” even though the client is emaciated. Which is the appropriate outcome criterion for this client’s disorder?
A. The client will consume adequate calories to sustain normal weight.
B. The client will cease strenuous exercise programs
C. The client will perceive personal ideal body weight and shape as normal
D. The client will not express a preoccupation with food.
Answer:
C. The client will perceive personal ideal body weight and shape as normal
(question is asking what appropriate outcome will be, and the question states that pt has altered body image. good outcome would be seeing body weight as normal.)

When counseling a client diagnosed with bulimia nervosa, a nurse explains that the client’s teeth will deteriorate because:
A. The emesis produced during purging is acidic and corrodes the tooth enamel.
B. Purging causes the depletion of dietary calcium.
C. Food is rapidly ingested without proper mastication.
D. Poor dental and oral hygiene leads to dental caries.
Answer:
A. The emesis produced during purging is acidic and corrodes the tooth enamel.

A nurse should explain to a client diagnosed with an eating disorder that behavior-modification programs are the treatment of choice because these programs:
A. Help the client correct a distorted body image
B. Address the underlying client anger
C. Manage the clients uncontrollable behaviors
D. Allow clients to maintain control
Answer:
D. Allow clients to maintain control

A high school senior is diagnosed with anorexia nervosa and is hospitalized for severe malnutrition. Her treatment team is planning to use behavior modification. What rationale should a nurse identify as the reasoning behind this therapy choice?
A. This therapy will increase the client’s motivation to gain weight.
B. This therapy will reward the client for perfectionist achievements.
C. This therapy will provide the client with control over behavioral choices.
D. This therapy will protect the client from parental overindulgence.
Answer:
C. This therapy will provide the client with control over behavioral choices.

A potential Olympic figure skater collapses during practice and is hospitalized for severe malnutrition. Anorexia nervosa is diagnosed. Which client statement best reflects insight related to this disorder?
A. Skaters need to be thin to improve their daily performance
B. All the skaters on the team are following an approved 1,200 calorie diet
C. The exercise of skating reduces my appetite but improves my energy level
D. I am angry at my mother. I can only get her approval when I win competitions.
D. I am angry at my mother. I can only get her approval when I win competitions.
(pt feels like they have no control)

The family of a client diagnosed with anorexia nervosa becomes defensive when the treatment team calls for a family meeting. Which is the appropriate nursing response?
A. Tell me why this family meeting is causing you to be defensive. All clients are required to participate in two family sessions.
B. Eating disorders have been correlated to certain familial patterns; without addressing these, your child’s condition will not improve.
C. Family dynamics are not linked to eating disorders. The meeting is to provide your child with family support.
D. Clients diagnosed with anorexia nervosa are part of the family system, and any alteration in family processes needs to be addressed.
Answer:
B. Eating disorders have been correlated to certain familial patterns; without addressing these, your child’s condition will not improve.
(A. never ask a pt why. D. only the bad processes need to be addressed.)

A client diagnosed with bulimia nervosa has been attending a mental health clinic for several months. Which factor should a nurse identify as an appropriate indicator of a positive client behavioral change?
A. The client gained 2 pounds in 1 wk
B. The client focused conversations on nutritious food
C. The client demonstrated healthy coping mechanisms that decreased anxiety
D. The client verbalized an understanding of the etiology of the disorder
Answer:
C. The client demonstrated healthy coping mechanisms that decreased anxiety

A morbidly obese client is prescribed an anorexiant medication. About which medication should a nurse teach the client?
A. Diazepam (Valium)
B. Dexfenfluramine (Redux)
C. Sibutramine (Meridia)
D. Pemoline (Cylert)
Answer:
C. Sibutramine (Meridia)
(helps to curve pt’s appetite. A. is a sedative. B. Increasees serotonin in pt’s brain and decreases urge to eat. D. CNS stimulant.)

A nurse is attempting to differentiate between the symptoms of anorexia nervosa and the symptoms of bulimia. which statement should the nurse identify as correct?
A. Clients diagnosed with anorexia nervosa experience extreme nutritional deficits, whereas clients diagnosed with bulimia nervosa do not.
B. Clients diagnosed with bulimia nervosa experience amenorrhea, whereas clients diagnosed with anorexia nervosa do not.
C. Clients diagnosed with bulimia nervosa experience hypotension, edema, and lanugo, whereas clients diagnosed with anorexia nervosa do not.
D. Clients diagnosed with anorexia nervosa have eroded tooth enamel, whereas clients diagnosed with bulimia nervosa do not.
A. Clients diagnosed with anorexia nervosa experience extreme nutritional deficits, whereas clients diagnosed with bulimia nervosa do not.

A client diagnosed with a history of anorexia nervosa comes to an outpatient clinic after being medically cleared. The client states, “My parents watch me like a hawk and never let me out of their sight. ” Which nursing diagnosis would take priority at this time?
A. Altered nutrition less than body requirements
B. Altered social interaction
C. Impaired verbal communication
D. Altered family processes
Answer:
D. Altered family processes

A nurse should identify topiramate (Topamax) as the drug of choice for which conditions? (select all that apply)
A. Binge eating with a diagnosis of obesity
B. Bingeing and purging with a diagnosis of bulimia nervosa.
C. Weight loss with a diagnosis of anorexia nervosa
D. Amenorrhea with a diagnosis of anorexia nervosa
E. Emaciation with a diagnosis of bulimia nervosa.
Answer:
A., B.
(Topamax is an anticonvulsant, but decreases urge to binge eat.)

What should be the priority nursing diagnosis for a client experiencing alcohol withdrawal?
A. risk for injury r/t CNS stimulation
B. Disturbed thought processes r/t tactile hallucinations
C. Ineffective coping r/t powerlessness over alcohol use.
D. Ineffective denial r/t continued alcohol use despite negative consequences
Answer:
A. risk for injury r/t CNS stimulation

A nurse evaluates a client’s patient-controlled analgesia pump and notices 100 attempts within a 30-minute period. Which is the best rationale for assessing this client for substance dependence?
A. Narcotic pain medication is contraindicated for all clients with active substance-abuse problems.
B. Clients who are dependent on alcohol or benzodiazepines may have developed cross-tolerance to analgesics and require increased doses to achieve effective pain control.
C. There is no need to assess the client for substance dependence. There is an obvious PCA malfunction, because these clients have a higher pain tolerance.
D. The client is experiencing symptoms of withdrawal and needs to be accurately assessed for lorazepam (Ativan) dosage.
Answer:
B. Clients who are dependent on alcohol or benzodiazepines may have developed cross-tolerance to analgesics and require increased doses to achieve effective pain control.

On the first day of a client’s alcohol detoxification, which nursing intervention should take priority?
A. Strongly encourage the client to attend 90 Alcoholics Anonymous meetings in 90 days.
B. Educate the client about the biopsychosocial consequences of alcohol abuse.
C. Administer ordered chlordiazepoxide (Librium) in a dosage according to protocol.
D. Administer vitamin B1 to prevent Wernicke-Korsakoff syndrome.
Answer:
C. Administer ordered chlordiazepoxide (Librium) in a dosage according to protocol.
(Librium is given to prevent adverse effects from happening)

Which client statement indicate a knowledge deficit related to substance abuse?
A. Although its legal, alcohol is one of the most widely abused drugs in our society.
B. Tolerance to heroin develops quickly.
C. Flashbacks from LSD use may reoccur spontaneously.
D. Marijuana is like smoking cigarettes. Everyone does it. It’s essentially harmless.
Answer:
D. Marijuana is like smoking cigarettes. Everyone does it. It’s essentially harmless.

We have an expert-written solution to this problem!
A client with a history of heavy alcohol use is brought to an emergency department by family members who state that the client has had nothing to drink in the last 24 hours. When the nurse reports to the ED physician, which client symptom should be the nurse’s first priority?
A. Tactile hallucinations
B. BP of 180/100 mm Hg
C. Mood rating of 2/10 on numeric scale
D. Dehydration
Answer:
B. BP of 180/100 mm Hg

Upon admission to an inpatient treatment facility for symptoms of alcohol withdrawal, a client states, “I haven’t eaten in 3 days.” A nurse’s assessment reveals BP 170/100 mm HG, P 110, R 28, and T 97F (36C) with dry skin, dry mucous membranes, and poor skin turgor. What should be the priority nursing diagnosis?
A. Knowledge deficit
B. Fluid volume excess
C. Imbalanced nutrition: Less than body requirements
D. Ineffective individual coping
Answer:
C. Imbalanced nutrition: Less than body requirements
(priority is always going to be what is keeping that pt alive. Physiological first.)

A mother who has a history of chronic heroin use has lost custody of her children due to abuse and neglect. She has been admitted to an inpatient substance abuse-program. Which client statement should a nurse associate with a positive prognosis for this client?
A. Im not going to use heroin ever again. I know I’ve got the willpower to do it this time.
I cannot control my use of heroin. Its stronger than I am.
C. Im going to get all my children back. they need their mother.
D. Once I deal with my childhood physical abuse, recovery should be easy.
Answer:

At the first meeting of a group of older adults at a daycare center for the elderly, the nurse asks one of the members what kinds of things she would like to do with the group. The older woman shrugs her shoulders and says, “You tell me, you’re the leader.” What is the best response for the nurse to make?

A. “Yes, I am the leader today. Would you like to be the leader tomorrow?”

B. “Yes, I will be leading this group. What would you like to accomplish during this time?”

C. “Yes, I have been assigned to be the leader of this group. I will be here for the next six weeks.”

D. “Yes, I am the leader. You seem angry about not being the leader yourself.”

B. “Yes, I will be leading this group. What would you like to accomplish during this time?”

Anxiety over participation in a group and testing of the leader characteristically occur in the initial phase of group dynamics.
(B) provides information and focuses the group back to defining its function.
(A) is manipulative bargaining.
Although (C) provides information, it does not focus the group on its purpose or task.
(D) is interpreting the client’s feelings and is almost challenging.

Over a period of several weeks, one male participant of a socialization group at a community day care center for the elderly monopolizes most of the group’s time and interrupts others when they are talking. What is the best action for the nurse to take in this situation?

A. Talk to the client outside the group about his behavior during group meetings.
B. Remind the client to allow others in the group a chance to talk.
C. Allow the group to handle the problem.
D. Ask the client to join another group.
C. Allow the group to handle the problem.

After several weeks, the group is in the working phase and the group members should be allowed to determine the direction of the group. The nurse should ignore the client’s comments and allow the group to handle the situation (C). A good leader should not have separate meetings with group members (A), as such behavior is manipulative on the part of the leader. (B) is dictatorial and is not in keeping with good leadership skills. (D) is avoiding the problem. Remember, identify what phase the group is in–initial, working, or termination–this will help determine communication style.

An 86-year-old female client with Alzheimer’s disease is wandering the busy halls of the extended care facility and asks the nurse, “Where should I stand for the parade?” Which response is best for the nurse to provide?
A. “Anywhere you want to stand as long as you do not get hurt by those in the parade.”
B. “You are confused because of all the activity in the hall. There is no parade.”
C. “Let’s go back to the activity room and see what is going on in there.”
D. “Remember I told you that this is a nursing home and I am your nurse.”
C. “Let’s go back to the activity room and see what is going on in there.”

It is common for those with Alzheimer’s disease to use the wrong words. Redirecting the client (using an accepting non-judgmental dialogue) to a safer place and familiar activities (C) is most helpful because clients experience short-term memory loss. (A) dismisses the client’s attempt to find order and does not help her relate to her surroundings. (B) dismisses the client and may increase her anxiety level because it merely labels the client’s behavior and offers no solution. It is very frustrating for those with Alzheimer’s disease to “remember,” and scolding them (D) may hurt their feelings.

Physical examination of a 6-year-old reveals several bite marks in various locations on his body. X-ray examination reveals healed fractures of the ribs. The mother tells the nurse that her child is always having accidents. Which initial response by the nurse is most appropriate?
A. “I need to inform the healthcare provider about your child’s tendency to be accident prone.”
B. “Tell me more specifically about your child’s accidents.”
C. “I must report these injuries to the authorities because they do not seem accidental.”
D. “Boys this age always seem to require more supervision and can be quite accident prone.”
B. “Tell me more specifically about your child’s accidents.”

(B) seeks more information using an open ended, non-threatening statement. (A) could be appropriate, but it is not the best answer because the nurse is being somewhat sarcastic and is also avoiding the situation by referring it to the healthcare provider for resolution. Although it is true that suspected cases of child abuse must be reported, (C) is virtually an attack and is jumping to conclusions before conclusive data has been obtained. (D) is a cliché and dismisses the seriousness of the situation.

A child is brought to the emergency room with a broken arm. Because of other injuries, the nurse suspects the child may be a victim of abuse. When the nurse tries to give the child an injection, the child’s mother becomes very loud and shouts, “I won’t leave my son! Don’t you touch him! You’ll hurt my child!” What is the best interpretation of the mother’s statements? The mother is
A. regressing to an earlier behavior pattern.
B. sublimating her anger.
C. projecting her feelings onto the nurse.
D. suppressing her fear.
C. projecting her feelings onto the nurse.

Projection is attributing one’s own thoughts, impulses, or behaviors onto another–it is the mother who is probably harming the child and she is attributing her actions to the nurse (C). The mother may be immature, but (A) is not the best description of her behavior. (B) is substituting a socially acceptable feeling for an unacceptable one. These are not socially acceptable feelings. The mother may be suppressing her fear (D) by displaying anger, but such an interpretation cannot be concluded from the data presented.

A 38-year-old female client is admitted with a diagnosis of paranoid schizophrenia. When her tray is brought to her, she refuses to eat and tells the nurse, “I know you are trying to poison me with that food.” Which response would be most appropriate for the nurse to make?
A. “I’ll leave your tray here. I am available if you need anything else.”
B. “You’re not being poisoned. Why do you think someone is trying to poison you?”
C. “No one on this unit has ever died from poisoning. You’re safe here.”
D. “I will talk to your healthcare provider about the possibility of changing your diet.”
A. “I’ll leave your tray here. I am available if you need anything else.”

(A) is the best choice cited. The nurse does not argue with the client nor demand that she eat, but offers support by agreeing to “be there if needed”, e. g., to warm the food. (B and C) are arguing with the client’s delusions, and (B) asks “why” which is usually not a good question for a psychotic client. (D) has nothing to do with the actual problem; i. e., the problem is not the diet (she thinks any food given to her is poisoned).

A 25-year-old female client has been particularly restless and the nurse finds her trying to leave the psychiatric unit. She tells the nurse, “Please let me go! I must leave because the secret police are after me.” Which response is best for the nurse to make?
A. “No one is after you, you’re safe here.”
B. “You’ll feel better after you have rested.”
C. “I know you must feel lonely and frightened.”
D. “Come with me to your room and I will sit with you.”
D. “Come with me to your room and I will sit with you.”

(D) is the best response because it offers support without judgment or demands. (A) is arguing with the client’s delusion. (B) is offering false reassurance. (C) is a violation of therapeutic communication in that the nurse is telling the client how she feels (frightened and lonely), rather than allowing the client to describe her own feelings. Hallucinating and/or delusional clients are not capable of discussing their feelings, particularly when they perceive a crisis.

A 45-year-old male client tells the nurse that he used to believe that he was Jesus Christ, but now he knows he is not. Which response is best for the nurse to make?
A. “Did you really believe you were Jesus Christ?”
B. “I think you’re getting well.”
C. “Others have had similar thoughts when under stress.”
D. “Why did you think you were Jesus Christ?”
C. “Others have had similar thoughts when under stress.”

(C) offers support by assuring the client that others have suffered as he has (also the principle on which Alcoholics Anonymous acts). (A) is belittling. (B) is making an inappropriate judgment. You may have narrowed your choices to (C and D). However, you should eliminate (D) because it is a “why” question, and the client does not know why!

A nurse working in the emergency room of a children’s hospital admits a child whose injuries could have resulted from abuse. Which statement most accurately describes the nurse’s responsibility in cases of suspected child abuse?
A. The nurse should obtain objective data such as x-rays before reporting suspicions to the authorities.
B. The nurse should confirm any suspicions of child abuse with the healthcare provider before reporting to the authorities.
C. The nurse should report any case of suspected child abuse to the nurse in charge.
D. The nurse should note in the client’s record any suspicions of child abuse so that a history of such suspicions can be tracked.
C. The nurse should report any case of suspected child abuse to the nurse in charge.

It is the nurse’s legal responsibility to report all suspected cases of child abuse. Notifying the charge nurse starts the legal reporting process (C).

A client who is being treated with lithium carbonate for bipolar disorder develops diarrhea, vomiting, and drowsiness. What action should the nurse take?
A. Notify the healthcare provider immediately and prepare for administration of an antidote.
B. Notify the healthcare provider of the symptoms prior to the next administration of the drug.
C. Record the symptoms as normal side effects and continue administration of the prescribed dosage.
D. Hold the medication and refuse to administer additional amounts of the drug.
B. Notify the healthcare provider of the symptoms prior to the next administration of the drug.

Early side effects of lithium carbonate (occurring with serum lithium levels below 2.0 mEq per liter) generally follow a progressive pattern beginning with diarrhea, vomiting, drowsiness, and muscular weakness. At higher levels, ataxia, tinnitus, blurred vision, and large dilute urine output may occur. (B) is the best choice. Although these are expected symptoms, the healthcare provider should be notified prior to the next administration of the drug. (A, C, and D) would not reflect good nursing judgment.

A client on the psychiatric unit appears to imitate a certain nurse on the unit. The client seeks out this particular nurse and imitates the nurse’s mannerisms. The nurse knows that the client is using which defense mechanism?
A. Sublimation.
B. Identification.
C. Introjection.
D. Repression.
B. Identification.

Identification (B) is an attempt to be like someone or emulate the personality traits of another. (A) is substituting an unacceptable feeling for one that is more socially acceptable. (C) is incorporating the values or qualities of an admired person or group into one’s own ego structure. (D) is the involuntary exclusion of painful thoughts or memories from one’s awareness.

The nurse is planning the care for a 32-year-old male client with acute depression. Which nursing intervention would be best in helping this client deal with his depression?
A. Ensure that the client’s day is filled with group activities.
B. Assist the client in exploring feelings of shame, anger, and guilt.
C. Allow the client to initiate and determine activities of daily living.
D. Encourage the client to explore the rationale for his depression.
B. Assist the client in exploring feelings of shame, anger, and guilt.

Depression is associated with feelings of shame, anger, and guilt. Exploring such feelings is an important nursing intervention for the depressed client (B). If the client’s day is filled with group activities (A) he might not have the opportunity to explore these feelings. (C) is a good intervention for the chronically depressed client who exhibits vegetative signs of depression. (D) is essentially asking the client “why” he is depressed–avoid “whys” disguised as “rationale.”

An anxious client expressing a fear of people and open places is admitted to the psychiatric unit. What is the most effective way for the nurse to assist this client?
A. Plan an outing within the first week of admission.
B. Distract her whenever she expresses her discomfort about being with others.
C. Confront her fears and discuss the possible causes of these fears.
D. Accompany her outside for an increasing amount of time each day.
D. Accompany her outside for an increasing amount of time each day.

The process of gradual desensitization by controlled exposure to the situation which is feared (D), is the treatment of choice in phobic reactions. (A and C) are far too aggressive for the initial treatment period and could even be considered hostile. (B) promotes denial of the problem, and gives the client the message that discussion of the phobia is not permitted.

A client with bipolar disorder on the mental health unit becomes loud, and shouts at one of the nurses, “You fat tub of lard! Get something done around here!” What is the best initial action for the nurse to take?
A. Have the orderly escort the client to his room.
B. Tell the client his healthcare provider will be notified if he continues to be verbally abusive.
C. Redirect the client’s energy by asking him to tidy the recreation room.
D. Call the healthcare provider to obtain a prescription for a sedative.
C. Redirect the client’s energy by asking him to tidy the recreation room.

Distracting the client, or redirecting his energy (C), prevents further escalation of the inappropriate behavior. (A) could result in escalating the abuse and unnecessarily involve another staff member in the abusive situation. (B) is a threat and is using a health team member (healthcare provider) as the threat. (D) may be indicated if the behavior escalates, but, at this time, the best initial action is (C).

A 35-year-old male client who has been hospitalized for two weeks for chronic paranoia continues to state that someone is trying to steal his clothing. The most appropriate action for the nurse to take is to
A. encourage the client to actively participate in assigned activities on the unit.
B. place a lock on the client’s closet.
C. ignore the client’s paranoid ideation to extinguish these behaviors.
D. explain to the client that his suspicions are false.
A. encourage the client to actively participate in assigned activities on the unit.

Diverting the client’s attention from paranoid ideation and encouraging him to complete assignments can be helpful in assisting him to develop a positive self-image (A). The client’s problem is not security, and (B) actually supports his paranoid ideation. (C) is not correct because ignoring the client’s symptoms may lower his self-esteem. The nurse should not argue with the client about his delusions (D), and should not try to reason with the client regarding his paranoid ideation.

On admission, a highly anxious client is described as delusional. The nurse understands that delusions are most likely to occur with which class of disorder?
A. Neurotic.
B. Personality.
C. Anxiety.
D. Psychotic.
D. Psychotic.

Delusions are false beliefs associated with psychotic behavior, and psychotic persons are not in touch with reality (D). (A, B, and C) are mental health disorders which are not associated with a break in reality, nor with hallucinations (false sensations such as hearing, or seeing) or delusions (false beliefs).

A client is admitted with a diagnosis of depression. The nurse knows that which characteristic is most indicative of depression?
A. Grandiose ideation.
B. Self-destructive thoughts.
C. Suspiciousness of others.
D. A negative view of self and the future.
D. A negative view of self and the future.

Negative self-image and feelings of hopelessness about the future (D) are specific indicators for depression. (A and/or C) occurs with paranoia or paranoid ideation. (B) may be seen in depressed clients, but are not always present, so (D) is a better answer than (B).

A 45-year-old female client is admitted to the psychiatric unit for evaluation. Her husband states that she has been reluctant to leave home for the last six months. The client has not gone to work for a month and has been terminated from her job. She has not left the house since that time. This client is displaying symptoms of what condition?
A. Claustrophobia.
B. Acrophobia.
C. Agoraphobia.
D. Post-traumatic stress disorder.
C. Agoraphobia.

Agoraphobia (C) is the fear of crowds or being in an open place. (A) is the fear of being in closed places. (B) is the fear of high places. Remember, a phobia is an unrealistic fear which is associated with severe anxiety. (D) consists of the development of anxiety symptoms following a life event that is particularly serious and stressful (war, witnessing a child killed, etc.) and is experienced with terror, fear, and helplessness–a phobia is different.

A client who has been admitted to the psychiatric unit tells the nurse, “My problems are so bad that no one can help me.” Which response is best for the nurse to make?
A. “How can I help?”
B. “Things probably aren’t as bad as they seem right now.”
C. “Let’s talk about what is right with your life.”
D. “I hear how miserable you are, but things will get better soon.”
A. “How can I help?”

Offering self-shows empathy and caring (A), and is the best of the choices provided. Combining the first part of (D) with (A) would be the best response, but this is not a fill-in-the-blank or an essay test! Choose the best of those choices provided and move on. (B) dismisses the client, things are bad as far as this client is concerned. (C) avoids the client’s problems and promotes denial. “I hear how miserable you are” is an example of reflective dialogue and would be the best choice if it were not for the rest of the sentence–“but things will get better” which is offering false reassurance.

A woman brings her 48-year-old husband to the outpatient psychiatric unit and describes his behavior to the admitting nurse. She states that he has been sleepwalking, cannot remember who he is, and exhibits multiple personalities. The nurse knows that these behaviors are often associated with
A. dissociative disorder.
B. obsessive-compulsive disorder.
C. panic disorder.
D. post-traumatic stress syndrome.
A. dissociative disorder.

Sleepwalking, amnesia, and multiple personalities are examples of detaching emotional conflict from one’s consciousness, which is the definition of a dissociative disorder (A). (B) is characterized by persistent, recurrent intrusive thoughts or urges (obsessions) that are unwilled and cannot be ignored, and provoke impulsive acts (compulsions) such as handwashing. (C) is an acute attack of anxiety characterized by personality disorganization. (D) is re-experiencing a psychologically terrifying or distressing event that is outside the usual range of human experience, such as war, rape, etc.

A 27-year-old female client is admitted to the psychiatric hospital with a diagnosis of bipolar disorder, manic phase. She is demanding and active. Which intervention should the nurse include in this client’s plan of care?
A. Schedule her to attend various group activities.
B. Reinforce her ability to make her own decisions.
C. Encourage her to identify feelings of anger.
D. Provide a structured environment with little stimuli.
D. Provide a structured environment with little stimuli.

Clients in the manic phase of a bipolar disorder require decreased stimuli and a structured environment (D). Plan noncompetitive activities that can be carried out alone. (A) is contraindicated; stimuli should be reduced as much as possible. Impulsive decision-making is characteristic of clients with bipolar disorder. To prevent future complications, the nurse should monitor these clients’ decisions and assist them in the decision-making process (B). (C) is more often associated with depression than with bipolar disorder.

The nurse plans to help an 18-year-old female intellectually disabled client ambulate the first postoperative day after an appendectomy. When the nurse tells the client it is time to get out of bed, the client becomes angry and tells the nurse, “Get out of here! I’ll get up when I’m ready!” Which response is best for the nurse to make?
A. “Your healthcare provider has prescribed ambulation on the first postoperative day.”
B. “You must ambulate to avoid complications which could cause more discomfort than ambulating.”
C. “I know how you feel. You’re angry about having to ambulate, but this will help you get well.”
D. “I’ll be back in 30 minutes to help you get out of bed and walk around the room.”
D. “I’ll be back in 30 minutes to help you get out of bed and walk around the room.”

(D) provides a “cooling off” period, is firm, direct, non-threatening, and avoids arguing with the client. (A) is avoiding responsibility by referring to the healthcare provider. (B) is trying to reason with an intellectually disabled client and is threatening the client with “complications.” (C) is telling the client how she feels (angry), and the nurse does not really “know” how this client feels, unless the nurse is also intellectually disabled and has also just had an appendectomy.

A 46-year-old female client has been on antipsychotic neuroleptic medication for the past three days. She has had a decrease in psychotic behavior and appears to be responding well to the medication. On the fourth day, the client’s blood pressure increases, she becomes pale and febrile, and demonstrates muscular rigidity. Which action will the nurse initiate?
A. Place the client on seizure precautions and monitor carefully.
B. Immediately transfer the client to ICU.
C. Describe the symptoms to the charge nurse and record on the client’s chart.
D. No action is required at this time as these are known side effects of such drugs.
B. Immediately transfer the client to ICU.

These symptoms are descriptive of neuroleptic malignant syndrome (NMS) which is an extremely serious/life threatening reaction to neuroleptic drugs (B). The major symptoms of this syndrome are fever, rigidity, autonomic instability, and encephalopathy. Respiratory failure, cardiovascular collapse, arrhythmias, and/or renal failure can result in death. This is an EMERGENCY situation, and the client requires immediate critical care. Seizure precautions (A) are not indicated in this situation. (C and D) do not consider the seriousness of the situation.

A male client is admitted to the psychiatric unit with a medical diagnosis of paranoid schizophrenia. During the admission procedure, the client looks up and states, “No, it’s not MY fault. You can’t blame me. I didn’t kill him, you did.” What action is best for the nurse to take?
A. Reassure the client by telling him that his fear of the admission procedure is to be expected.
B. Tell the client that no one is accusing him of murder and remind him that the hospital is a safe place.
C. Assess the content of the hallucinations by asking the client what he is hearing.
D. Ignore the behavior and make no response at all to his delusional statements.
C. Assess the content of the hallucinations by asking the client what he is hearing.

Further assessment is indicated (C). The nurse should obtain information about what the client believes the voices are telling him–they may be telling him to kill the nurse! (A) is telling the client how he feels (fearful). The nurse should leave communications open and seek more information. (B) is arguing with the client’s delusion, and the nurse should never argue with a client’s hallucinations or delusions, also (B) is possibly offering false reassurance. (D) is avoiding the situation and the client’s needs.

A 35-year-old male client on the psychiatric unit of a general hospital believes that someone is trying to poison him. The nurse understands that a client’s delusions are most likely related to his
A. early childhood experiences involving authority issues.
B. anger about being hospitalized.
C. low self-esteem.
D. phobic fear of food.
C. low self-esteem.

Psychotic clients have difficulty with trust and have low self-esteem (C). Nursing care should be directed at building trust and promoting positive self-esteem. Activities with limited concentration and no competition should be encouraged in order to build self-esteem. (A, B, and D) are not specifically related to the development of delusions.

A client who is diagnosed with schizophrenia is admitted to the hospital. The nurse assesses the client’s mental status. Which assessment finding is most characteristic of a client with schizophrenia?
A. Mood swings.
B. Extreme sadness.
C. Manipulative behavior.
D. Flat affect.
D. Flat affect.

Disinterest, and diminished or lack of facial expression is characteristic of schizophrenia and is referred to as a flat affect (D). (A) is associated with bipolar disorder. (B) is associated with depression. (C) is usually associated with personality disorders and is often seen in clients who abuse substances.

The nurse is conducting discharge teaching for a client with schizophrenia who plans to live in a group home. Which statement is most indicative of the need for careful follow-up after discharge?
A. “Crickets are a good source of protein.”
B. “I have not heard any voices for a week.”
C. “Only my belief in God can help me.”
D. “Sometimes I have a hard time sitting still.”
C. “Only my belief in God can help me.”

The most frequent cause of increased symptoms in psychotic clients is non-compliance with the medication regimen. If clients believe that “God alone” is going to heal them (C), then they may discontinue their medication, so (C) would pose the greatest threat to this client’s prognosis. (A) would require further teaching, but is not as significant a statement as (C). (B) indicates an improvement in the client’s condition. (D) may be a sign of anxiety that could improve with treatment, but does not have the priority of (C).

A 52-year-old male client in the intensive care unit who has been oriented suddenly becomes disoriented and fearful. Assessment of vital signs and other physical parameters reveal no significant change and the nurse formulates the diagnosis, “Confusion related to ICU psychosis.” Which intervention would be best to implement?
A. Move all machines away from the client’s immediate area.
B. Attempt to allay the client’s fears by explaining the etiology of his condition.
C. Cluster care so that brief periods of rest can be scheduled during the day.
D. Extend visitation times for family and friends.
C. Cluster care so that brief periods of rest can be scheduled during the day.

The critical care environment confronts clients with an environment which provides stressors heightened by treatment modalities that may prove to be lifesaving. These stressors can result in isolation and confusion. The best intervention is to provide the client with rest periods (C). (A) is not practical–the machinery is often lifesaving. The client is not ready for (B). Although family and friends (D) can provide a support system to the client, visits should be limited because of the critical care that must be provided.

The nurse observes a female client with schizophrenia watching the news on TV. She begins to laugh softly and says, “Yes, my love, I’ll do it.” When the nurse questions the client about her comment she states, “The news commentator is my lover and he speaks to me each evening. Only I can understand what he says.” What is the best response for the nurse to make?
A. “What do you believe the news commentator said to you?”
B. “Let’s watch news on a different television channel.”
C. “Does the news commentator have plans to harm you or others?”
D. “The news commentator is not talking to you.”
A. “What do you believe the news commentator said to you?”

It is imperative that the nurse determine what the client believes she heard (A). The idea of reference may be to hurt herself or someone else, and the main function of a psychiatric nurse is to maintain safety. (B) is acceptable, but it is best to determine the client’s beliefs. (C) is validating the idea of reference, while (D) is challenging the client.

A male client with schizophrenia who is taking fluphenazine decanoate (Prolixin decanoate) is being discharged in the morning. A repeat dose of medication is scheduled for 20 days after discharge. The client tells the nurse that he is going on vacation in the Bahamas and will return in 18 days. Which statement by the client indicates a need for health teaching?
A. “When I return from my tropical island vacation, I will go to the clinic to get my Prolixin injection.”
B. “While I am on vacation and when I return, I will not eat or drink anything that contains alcohol.”
C. “I will notify the healthcare provider if I have a sore throat or flu-like symptoms.”
D. “I will continue to take my benztropine mesylate (Cogentin) every day.”
A. “When I return from my tropical island vacation, I will go to the clinic to get my Prolixin injection.”

Photosensitivity is a side effect of Prolixin and a vacation in the Bahamas (with its tropical island climate) increases the client’s chance of experiencing this side effect. He should be instructed to avoid direct sun (A) and wear sunscreen. (B, C, and D) indicate accurate knowledge. Alcohol acts synergistically with Prolixin (B). (C) lists signs of agranulocytosis, which is also a side effect of Prolixin. In order to avoid extrapyramidal symptoms (EPS), anticholinergic drugs, such as Cogentin, are often prescribed prophylactically with Prolixin.

The nurse is leading a “current events group” with chronic psychiatric clients. One group member states, “Clara Barton was my nurse during my last hospitalization. She was a very mean nurse and wasn’t nice to me.” Which response would be best for the nurse to make?
A. “Clara Barton was not your nurse.”
B. “What did she do to you that was so mean?”
C. “I didn’t know that Clara Barton was a nurse.”
D. “Clara Barton started the American Red Cross.”
D. “Clara Barton started the American Red Cross.”

(D) presents the reality of the situation in relation to American culture. The fact that Clara Barton was a nurse during the Civil War should be addressed on an individual basis. Since this is group therapy, the nurse would be illustrating the concept of universality. (A) is likely to promote defensiveness. (B and C) would support the delusion.

Based on non-compliance with the medication regimen, an adult client with a medical diagnosis of substance abuse and schizophrenia was recently switched from oral fluphenazine HCl (Prolixin) to IM fluphenazine decanoate (Prolixin Decanoate). What is most important to teach the client and family about this change in medication regimen?
A. Signs and symptoms of extrapyramidal effects (EPS).
B. Information about substance abuse and schizophrenia.
C. The effects of alcohol and drug interaction.
D. The availability of support groups for those with dual diagnoses.
C. The effects of alcohol and drug interaction.

Alcohol enhances the EPS side effects of Prolixin. The half-life of Prolixin PO is 8 hours, whereas the half-life of the Prolixin Decanoate IM is 2 to 4 weeks. That means the side effects of drinking alcohol are far more severe when the client drinks alcohol after taking the long-acting Prolixin Decanoate IM. (A, B, and D) provide valuable information and should be included in the client/family teaching, but they do not have the priority of (C).

The nurse suspects child abuse when assessing a 3-year-old boy and noticing several small, round burns on his legs and trunk that might be the result of cigarette burns. Which parental behavior provides the greatest validation for such suspicions?
A. The parents’ explanation of how the burns occurred is different from the child’s explanation of how they occurred.
B. The parents seem to dismiss the severity of the child’s burns, saying they are very small and have not posed any problem.
C. The parents become very anxious when the nurse suggests that the child may need to be admitted for further evaluation.
D. The parents tell the nurse that the child was burned in a house fire which is incompatible with the nurse’s observation of the type of burn.
D. The parents tell the nurse that the child was burned in a house fire which is incompatible with the nurse’s observation of the type of burn.

(D) provides the most validation. The parent’s explanation (subjective data) is incompatible with the objective data (small round burns on the legs and trunk). (A) provides only subjective data, and the child’s explanation could be influenced by factors such as age, fear, or imagination. The parent’s apparent lack of concern (B) is inconclusive, but the nurse’s opinion of the parents’ reaction is subjective and could be wrong. (C) might provide a clue that child abuse occurred, but the nurse must remember that most parents are anxious about their child being hospitalized.

At a support meeting of parents of a teenager with polysubstance dependency, a parent states, “Each time my son tries to quit taking drugs, he gets so depressed that I’m afraid he will commit suicide.” The nurse’s response should be based on which information?
A. Addiction is a chronic, incurable disease.
B. Tolerance to the effects of drugs causes feelings of depression.
C. Feelings of depression frequently lead to drug abuse and addiction.
D. Careful monitoring should be provided during withdrawal from the drugs.
D. Careful monitoring should be provided during withdrawal from the drugs.

The priority is to teach the parents that their son will need monitoring and support during withdrawal (D) to ensure that he does not attempt suicide. Although (A and C) are true, they are not as relevant to the parent’s expressed concern. There is no information to support (B).

The parents of a 14-year-old boy bring their son to the hospital. He is lethargic, but responsive. The mother states, “I think he took some of my pain pills.” During initial assessment of the teenager, what information is most important for the nurse to obtain from the parents?
A. If he has seemed depressed recently.
B. If a drug overdose has ever occurred before.
C. If he might have taken any other drugs.
D. If he has a desire to quit taking drugs.
C. If he might have taken any other drugs.

Knowledge of all substances taken (C) will guide further treatment, such as administration of antagonists, so obtaining this information has the highest priority. (A and B) are also valuable in planning treatment. (D) is not appropriate during the acute management of a drug overdose.

The nurse should hold the next scheduled dose of a client’s haloperidol (Haldol) based on which assessment finding(s)?
A. Dizziness when standing.
B. Shuffling gait and hand tremors.
C. Urinary retention.
D. Fever of 102 F.
D. Fever of 102 F.

A fever (D) may indicate neuroleptic malignant syndrome (NMS), a potentially fatal complication of antipsychotics. The healthcare provider should be contacted before administering the next dose of Haldol. (A, B, and C) are all adverse effects of Haldol which can be managed.

A 65-year-old female client complains to the nurse that recently she has been hearing voices. What question should the nurse ask this client first?
A. “Do you have problems with hallucinations?”
B. “Are you ever alone when you hear the voices?”
C. “Has anyone in your family had hearing problems?”
D. “Do you see things that others cannot see?”
B. “Are you ever alone when you hear the voices?”
Determining if the client is alone when she hears voices (B) will assist in differentiating between hallucinations and hearing loss; this is especially important in the aging population. If the client is experiencing hallucinations, the voices will be real to her, and it is unlikely that (A) would provide accurate information. (C and D) might be good follow-up questions, but would not have the priority of (B).

The charge nurse is collaborating with the nursing staff about the plan of care for a client who is very depressed. What is the most important intervention to implement during the first 48 hours after the client’s admission to the unit?
A. Monitor appetite and observe intake at meals.
B. Maintain safety in the client’s milieu.
C. Provide ongoing, supportive contact.
D. Encourage participation in activities.
B. Maintain safety in the client’s milieu.

The most important reason for closely observing a depressed client immediately after admission is to maintain safety (B), since suicide is a risk with depression. (A, C, and D) are all important interventions, but safety is the priority.

Within several days of hospitalization, a client is repeatedly washing the top of the same table. Which initial intervention is best for the nurse to implement to help the client cope with anxiety related to this behavior?
A. Administer a prescribed PRN antianxiety medication.
B. Assist the client to identify stimuli that precipitates the ritualistic activity.
C. Allow time for the ritualistic behavior, then redirect the client to other activities.
D. Teach the client relaxation and thought stopping techniques.
C. Allow time for the ritualistic behavior, then redirect the client to other activities.

Initially, the nurse should allow time for the ritual (C) to prevent anxiety. (A) may help reduce the client’s anxiety, but will not prevent ritualistic behavior resulting from the client’s ineffective coping ability. (B) is a long-term goal of individual therapy, but is not directly related to controlling the behavior at this time. (D) lists techniques that can be used to assist the client in learning new ways of interrupting obsessive thoughts and resulting ritualistic behavior as treatment progresses.

A female client with depression attends group and states that she sometimes misses her medication appointments because she feels very anxious about riding the bus. Which statement is the nurse’s best response?
A. “Can your case manager take you to your appointments?”
B. “Take your medication for anxiety before you ride the bus.”
C. “Let’s talk about what happens when you feel very anxious.”
D. “What are some ways that you can cope with your anxiety?”
D. “What are some ways that you can cope with your anxiety?”

The best response is to explore ways for the client to cope with anxiety (D). The nurse should encourage problem-solving rather than dependence on the case manager (A) for transportation. Strategies for coping with anxiety should be encouraged before suggesting (B). (C) is therapeutic, but the best response is an open-ended question to explore ways to cope with the anxiety.

A nurse working on a mental health unit receives a community call from a person who is tearful and states, “I just feel so nervous all of the time. I don’t know what to do about my problems. I haven’t been able to sleep at night and have hardly eaten for the past 3 or 4 days.” The nurse should initiate a referral based on which assessment?
A. Altered thought processes.
B. Moderate levels of anxiety.
C. Inadequate social support.
D. Altered health maintenance.
B. Moderate levels of anxiety.

The nurse should initiate a referral based on anxiety levels (B) and feelings of nervousness that interfere with sleep, appetite, and the inability to solve problems. The client does not report symptoms of (A) or evidence of (C). There is not enough information to initiate a referral based on (D).

A female client refuses to take an oral hypoglycemic agent because she believes that the drug is being administered as part of an elaborate plan by the Mafia to harm her. Which nursing intervention is most important to include in this client’s plan of care?
A. Reassure the client that no one will harm her while she is in the hospital.
B. Ask the healthcare provider to give the client the medication.
C. Explain that the diabetic medication is important to take.
D. Reassess client’s mental status for thought processes and content.
D. Reassess client’s mental status for thought processes and content.

The most important intervention is to reassess the client’s mental status (D) and to take further action based on the findings of this assessment. Attempting to reassure the client (A) is in effect arguing with the client’s delusions and could escalate an already anxious situation. Collaborating about diabetic care (B and C) is not likely to help change the client’s false beliefs.

The nurse is planning discharge for a male client with schizophrenia. The client insists that he is returning to his apartment, although the healthcare provider informed him that he will be moving to a boarding home. What is the most important nursing diagnosis for discharge planning?
A. Ineffective denial related to situational anxiety.
B. Ineffective coping related to inadequate support.
C. Social isolation related to difficult interactions.
D. Self-care deficit related to cognitive impairment.
A. Ineffective denial related to situational anxiety.

The best nursing diagnosis is (A) because the client is unable to acknowledge the move to a boarding home. (B, C, and D) are potential nursing diagnoses, but denial is most important because it is a defense mechanism that keeps the client from dealing with his feelings about living arrangements.

A male client is admitted to the mental health unit because he was feeling depressed about the loss of his wife and job. The client has a history of alcohol dependency and admits that he was drinking alcohol 12 hours ago. Vital signs are: temperature, 100 F, pulse 100, and BP 142/100. The nurse plans to give the client lorazepam (Ativan) based on which priority nursing diagnosis?
A. Risk for injury related to suicidal ideation.
B. Risk for injury related to alcohol detoxification.
C. Knowledge deficit related to ineffective coping.
D. Health seeking behaviors related to personal crisis.
B. Risk for injury related to alcohol detoxification.

The most important nursing diagnosis is related to alcohol detoxification (B) because the client has elevated vital signs, a sign of alcohol detoxification. Maintaining client safety related to (A) should be addressed after giving the client Ativan for elevated vital signs secondary to alcohol withdrawal. (C and D) can be addressed when immediate needs for safety are met.

An elderly female client with advanced dementia is admitted to the hospital with a fractured hip. The client repeatedly tells the staff, “Take me home. I want my Mommy.” Which response is best for the nurse to provide?
A. Orient the client to the time, place, and person.
B. Tell the client that the nurse is there and will help her.
C. Remind the client that her mother is no longer living.
D. Explain the seriousness of her injury and need for hospitalization.
B. Tell the client that the nurse is there and will help her.

Those with dementia often refer to home or parents when seeking security and comfort. The nurse should use the techniques of “offering self” and “talking to the feelings” to provide reassurance (B). Clients with advanced dementia have permanent physiological changes in the brain (plaques and tangles) that prevent them from comprehending and retaining new information, so (A, C, and D) are likely to be of little use to this client and do not help the client’s emotional needs.

When preparing a teaching plan for a client who is to be discharged with a prescription for lithium carbonate (Lithonate), it is most important for the nurse to include which instruction?
A. “It may take 3 to 4 weeks to achieve therapeutic effects.”
B. “Keep your dietary salt intake consistent.”
C. “Avoid eating aged cheese and chicken liver.”
D. “Eat foods high in fiber such as whole grain breads.”
B. “Keep your dietary salt intake consistent.”

Lithium’s effectiveness is influenced by salt intake (B). Too much salt causes more lithium to be excreted, thereby decreasing the effectiveness of the drug. Too little salt causes less lithium to be excreted, potentially resulting in toxicity. (A, C, and D) are not specific instructions pertinent to teaching about lithium carbonate (Lithonate).

The nurse is preparing to administer phenelzine sulfate (Nardil) to a client on the psychiatric unit. Which complaint related to administration of this drug would the nurse expect this client to make?
A. “My mouth feels like cotton.”
B. “That stuff gives me indigestion.”
C. “This pill gives me diarrhea.”
D. “My urine looks pink.”
A. “My mouth feels like cotton.”

A dry mouth (A) is an anticholinergic effect that is an expected side effect of MAO inhibitors such as phenelzine sulfate (Nardil). (B, C, and D) are not expected side effects of this medication.

A client is receiving substitution therapy during withdrawal from benzodiazepines. Which expected outcome statement has the highest priority when planning nursing care?
A. Client will not demonstrate cross-addiction.
B. Co-dependent behaviors will be decreased.
C. Excessive CNS stimulation will be reduced. D. Client’s level of consciousness will increase.
C. Excessive CNS stimulation will be reduced.

Substitution therapy with another CNS depressant is intended to decrease the excessive CNS stimulation that can occur during benzodiazepine withdrawal (C). (A, B, and D) are all appropriate outcome statements for the client described, but do not have the priority of (C).

A client who is known to abuse drugs is admitted to the psychiatric unit. Which medication should the nurse anticipate administering to a client who is exhibiting benzodiazepine withdrawal symptoms?
A. Perphenazine (Trilafon).
B. Diphenhydramine (Benadryl).
C. Chlordiazepoxide (Librium).
D. Isocarboxazid (Marplan).
C. Chlordiazepoxide (Librium).

Librium (C), an antianxiety drug, as well as other benzodiazepines, is used in titrated doses to reduce the severity of abrupt benzodiazepine withdrawal. (A) is an antipsychotic agent. (B) is an antihistamine and antianxiety drug. (D) is an MAO inhibitor.

5A 22-year-old male client is admitted to the emergency center following a suicide attempt. His records reveal that this is his third suicide attempt in the past two years. He is conscious, but does not respond to verbal commands for treatment. Which assessment finding should prompt the nurse to prepare the client for gastric lavage?
A. He ingested the drug 3 hours prior to admission to the emergency center.
B. The family reports that he took an entire bottle of acetaminophen (Tylenol).
C. He is unresponsive to instructions and is unable to cooperate with emetic therapy.
D. Those with repeated suicide attempts desire punishment to relieve their guilt.
C. He is unresponsive to instructions and is unable to cooperate with emetic therapy.

Because the client is unable to follow instructions, emetic therapy would be very difficult to implement and gastric lavage would be necessary (C). (A and B) should be considered in determining the course of treatment, but they are not the basis for determining if gastric lavage will be implemented. Medical treatments should never be used as “punitive” measures (D).

A 72-year-old female client is admitted to the psychiatric unit with a diagnosis of major depression. Which statement by the client should be of greatest concern to the nurse and require further assessment?
A. “I will die if my cat dies.”
B. “I don’t feel like eating this morning.”
C. “I just went to my friend’s funeral.”
D. “Don’t you have more important things to do?”
A. “I will die if my cat dies.”

Sometimes a client will use an analogy to describe themselves, and (A) would be an indication for conducting a suicide assessment. (B) could have a variety of etiologies, and while further assessment is indicated, this statement does not indicate potential suicide. Normal grief process differs from depression, and at this client’s age peer/cohort deaths are more frequent, so (C) would be within normal limits. (D) is an expression of low self-esteem typical of depression. (B, C, and D) are examples of decreased energy and mood levels which would negate suicide ideation at this time.

A 19-year-old female client with a diagnosis of anorexia nervosa wants to help serve dinner trays to other clients on a psychiatric unit. What action should the nurse take?
A. Encourage the client’s self-motivation by asking her to pass trays for the rest of the week.
B. Provide an additional challenge by asking the client to help feed the older clients.
C. Suggest another way for this client to participate in the unit’s activities.
D. Tell the client that hospital guidelines allow only staff to pass the trays.
C. Suggest another way for this client to participate in the unit’s activities.

Clients with anorexia should not be allowed to plan or prepare food for unit activities and their desires to do so should be redirected (C). These clients gain pleasure from providing others with food and watching them eat. Such behaviors reinforce their perception of self-control. (A and B) are contraindicated for a client with anorexia nervosa. (D) avoids addressing the problem, so it is best to suggest another activity for the client.

Which diet selection by a client who is depressed and taking the MAO inhibitor tranylcypromine sulfate (Parnate) indicates to the nurse that the client understands the dietary restrictions imposed by this medication regimen?
A. Hamburger, French fries, and chocolate milkshake.
B. Liver and onions, broccoli, and decaffeinated coffee.
C. Pepperoni and cheese pizza, tossed salad, and a soft drink.
D. Roast beef, baked potato with butter, and iced tea.
D. Roast beef, baked potato with butter, and iced tea.

Only (D) contains no tyramine. Tyramine in foods interacts with MAOI in the body causing a hypertensive crisis which is life-threatening, and Parnate is classified as an MAOI antidepressant. Some items in (A, B, and C) contain tyramine and would not be permitted for a client taking Parnate.

The nurse is planning care for a 32-year-old male client diagnosed with HIV infection who has a history of chronic depression. Recently, the client’s viral load has begun to increase rather than decrease despite his adherence to the HIV drug regimen. What should the nurse do first while taking the client’s history upon admission to the hospital?
A. Determine if the client attends a support group weekly.
B. Hold all antidepressant medications until further notice.
C. Ask the client if he takes St. John’s Wort routinely.
D. Have the client describe any recent changes in mood.
C. Ask the client if he takes St. John’s Wort routinely.

St. John’s Wort, an herbal preparation, is an alternative (nonconventional) therapy for depression, but it may adversely interact with medications used to treat HIV infection (C). The nurse’s top priority upon admission is to determine if the client has been taking this herb concurrently with HIV antiviral drugs, which may explain the rise in the viral load. Asking about (A or D) may be helpful in gathering more data about the client’s depressive state, but these issues do not have the priority of (C). (B) may be harmful to the client.

A young adult male client, diagnosed with paranoid schizophrenia, believes that world is trying poison him. What intervention should the nurse include in this client’s plan of care?
A. Remind the client that his suspicions are not true.
B. Ask one nurse to spend time with the client daily.
C. Encourage the client to participate in group activities.
D. Assign the client to a room closest to the activity room.
B. Ask one nurse to spend time with the client daily.

A client with paranoid schizophrenia has difficulty with trust and developing a trusting relationship with one nurse (B) is likely to be therapeutic for this client. (A) is argumentative. Stress increases anxiety, and anxiety increases paranoid ideation; (C) would be too stressful and anxiety-promoting for a client who is experiencing pathological suspicions. (D) also might increase anxiety and stress.

The community health nurse talks to a male client who has bipolar disorder. The client explains that he sleeps 4 to 5 hours a night and is working with his partner to start two new businesses and build an empire. The client stopped taking his medications several days ago. What nursing problem has the highest priority?
A. Excessive work activity.
B. Decreased need for sleep.
C. Medication management.
D. Inflated self-esteem.
C. Medication management.

The most important nursing problem is medication management (C) because compliance with the medication regimen will help prevent hospitalization. The client is also exhibiting signs of (A, B, and C); however, these problems do not have the priority of medication management.

A male client is admitted to a mental health unit on Friday afternoon and is very upset on Sunday because he has not had the opportunity to talk with the healthcare provider. Which response is best for the nurse to provide this client?
A. “Let me call and leave a message for your healthcare provider.”
B. “The healthcare provider should be here on Monday morning.”
C. “How can I help answer your questions?”
D. “What concerns do you have at this time?”
A. “Let me call and leave a message for your healthcare provider.”

It is best for the nurse to call the healthcare provider (A) because clients have the right to information about their treatment. Suggesting that the healthcare provider will be available the following day (B) does not provide immediate reassurance to the client. The nurse can also implement offer to assist the client (C and D), but the highest priority intervention is contacting the healthcare provider.

A male client with mental illness and substance dependency tells the mental health nurse that he has started using illegal drugs again and wants to seek treatment. Since he has a dual diagnosis, which person is best for the nurse to refer this client to first?
A. The emergency room nurse.
B. His case manager.
C. The clinic healthcare provider.
D. His support group sponsor.
B. His case manager.

The case manager (B) is responsible for coordinating community services, and since this client has a dual diagnosis, this is the best person to describe available treatment options. (A) is unnecessary, unless the client experiences behaviors that threaten his safety or the safety of others. (C and D) might also be useful, but it is most important at this time that a treatment program be coordinated to meet this client’s needs.

On admission to a residential care facility, an elderly female client tells the nurse that she enjoys cooking, quilting, and watching television. Twenty-fours after admission, the nurse notes that the client is withdrawn and isolated. It is best for the nurse to encourage this client to become involved in which activity?
A. Clean the unit kitchen cabinets.
B. Participate in a group quilting project.
C. Watch television in the activity room.
D. Bake a cake for a resident’s birthday.
B. Participate in a group quilting project.

Peer interaction in a group activity (B) will help to prevent social isolation and withdrawal. (A, C, and D) are activities that can be accomplished alone, without peer interaction.

The wife of a male client recently diagnosed with schizophrenia asks the nurse, “What exactly is schizophrenia? Is my husband all right?” Which response is best for the nurse to provide to this family member?
A. “It sounds like you’re worried about your husband. Let’s sit down and talk.”
B. “It is a chemical imbalance in the brain that causes disorganized thinking.”
C. “Your husband will be just fine if he takes his medications regularly.”
D. “I think you should talk to your husband’s psychologist about this question.”
B. “It is a chemical imbalance in the brain that causes disorganized thinking.”

The nurse should answer the client’s question with factual information and explain that schizophrenia is a chemical imbalance in the brain (B). (A) is a therapeutic response but does not answer the question, and may be an appropriate response after the nurse answers the question asked. Although (C) is likely true to some degree, it is also true that some clients continue to have disorganized thinking even with antipsychotic medications. Referring the spouse to the psychologist (D) is avoiding the issue; the nurse can and should answer the question.

A 40-year-old male client diagnosed with schizophrenia and alcohol dependence has not had any visitors or phone calls since admission. He reports he has no family that cares about him and was living on the streets prior to this admission. According to Erikson’s theory of psychosocial development, which stage is the client in at this time?
A. Isolation.
B. Stagnation.
C. Despair.
D. Role confusion.
B. Stagnation.

The client is in Erikson’s “Generativity vs. Stagnation” stage (age 24 to 45), and meeting the task includes maintaining intimate relationships and moving toward developing a family (B). (A) occurs in young adulthood (age 18 to 25), (C) occurs in maturity (age 45 to death), and (D) occurs in adolescence (age 12 to 20). These are all stages that occur if individuals are not successfully coping with their psychosocial developmental stage.

The nurse should include which interventions in the plan of care for a severely depressed client with neurovegetative symptoms? (Select all that apply.)
A. Permit rest periods as needed.
B. Speaking slowly and simply.
C. Place the client on suicide precautions.
D. Allow the client extra time to complete tasks.
E. Observe and encourage food and fluid intake.
F. Encourage mild exercise and short walks on the unit
A. Permit rest periods as needed.
B. Speaking slowly and simply.
D. Allow the client extra time to complete tasks.
E. Observe and encourage food and fluid intake.
F. Encourage mild exercise and short walks on the unit

(A, B, D, E, and F) should be included in this client’s plan of care because these measures promote the client’s comfort and well-being. Neurovegetative symptoms accompany the mood disorder of depression and include physiological disruptions, such as anorexia, constipation, sleep disturbance, and psychomotor retardation. Suicidal ideation (C) does not usually accompany the neurovegetative state because the client does not have the energy or high level of anxiety associated with a suicide attempt.

A woman arrives in the Emergency Center and tells the nurse she thinks she has been raped. The client is sobbing and expresses disbelief that a rape could happen because the man is her best friend. After acknowledging the client’s fear and anxiety, how should the nurse respond?
A. “I would be very upset and mad if my best friend did that to me.”
B. “You must feel betrayed, but maybe you might have led him on?”
C. “Rape is not limited to strangers and frequently occurs by someone who is known to the victim.”
D. “This does not sound like rape. Did you change your mind about having sex after the fact?”
C. “Rape is not limited to strangers and frequently occurs by someone who is known to the victim.”

A victim of date rape or acquaintance rape is less prone to recognize what is happening because the incident usually involves persons who know each other, and the dynamics are different than rape by a stranger. (C) provides confrontation for the client’s denial because the victim frequently knows and trusts the perpetrator. Nurses should not express personal feelings (A) when dealing with victims. Suggesting that the client led on the rapist (B) indicates that the sexual assault was somehow the victim’s fault. (D) is judgmental and does not display compassion or establish trust between the nurse and the client.

Which statement about contemporary mental health nursing practice is accurate?
A. There is one approved theoretical framework for psychiatric nursing practice.
B. Psychiatric nursing has yet to be recognized as a core mental health discipline.
C. Contemporary practice of psychiatric nursing is primarily focused on inpatient care.
D. The psychiatric nursing client may be an individual, family, group, organization, or community.
D. The psychiatric nursing client may be an individual, family, group, organization, or community.

Mental health nursing is not only concerned with one-on-one interactions. Psychiatric stressors can impact and be reflected in the overall direction, activities, and responses involving families, groups, and entire communities (D). (A, B, and C) are incorrect statements about the status of mental health nursing.

The nurse is taking a history for a female client who is requesting a routine female exam. Which assessment finding requires follow-up?
A. Menstruation onset at age 9.
B. Contraceptive method includes condoms only.
C. Menstrual cycle occurs every 35 days.
D. “Black-out” after one drink last night on a date.
D. “Black-out” after one drink last night on a date.

A “black-out” typically occurs after ingestion of alcohol beverages that the client has no recall of the experiences or one’s behavior and is indicative of high blood alcohol levels, but the client’s experience of a “black-out” after one drink (D) is suspicious of the client receiving a “date rape” drug (Flunitrazepam) and needs additional follow-up. Although (A and C) occur on the outer ranges of “average,” both are within acceptable or “normal” ranges. (B) is an individual preference, but using condoms as the only contraceptive method carries a higher chance of conception.

The nurse is assessing the parents of a nuclear family who are attending a support group for parents of adolescents. According to Erikson, these parents who are adapting to middle adulthood should exhibit which characteristic?
A. Loss of independence.
B. Increased self-understanding.
C. Isolation from society.
D. Development of intimate relationships.
B. Increased self-understanding.

Middle adulthood is characterized by self-reflection, understanding, and acceptance (B), and generativity or guidance of children. (A and C) are maladaptive behaviors in middle adulthood. Although middle-aged adults may delay or re-establish intimate relationships, (D) is initially developed during young adulthood.

The nurse observes a client who is admitted to the mental health unit and identifies that the client is talking continuously, using words that rhyme but that have no context or relationship with one topic to the next in the conversation. This client’s behavior and thought processes are consistent with which syndrome?
A. Dementia.
B. Depression.
C. Schizophrenia.
D. Chronic brain syndrome.
C. Schizophrenia.

The client is demonstrating symptoms of schizophrenia (C), such as disorganized speech that may include word salad (communication that includes both real and imaginary words in no logical order), incoherent speech, and clanging (rhyming). Dementia (A) is a global impairment of intellectual (cognitive) functions that may be progressive, such as Alzheimer’s or organic brain syndrome (D). Depression (C) is typified by psychomotor retardation, and the client appears to be slowed down in movement, in speech, and would appear listless and disheveled.

A female client with obsessive-compulsive disorder (OCD) is describing her obsessions and compulsions and asks the nurse why these make her feel safer. What information should the nurse include in this client’s teaching plan? (Select all that apply.)
A. Compulsions relieve anxiety.
B. Anxiety is the key reason for OCD.
C. Obsessions cause compulsions.
D. Obsessive thoughts are linked to levels of neurochemicals.
E. Antidepressant medications increase serotonin levels.
A. Compulsions relieve anxiety.
B. Anxiety is the key reason for OCD.
D. Obsessive thoughts are linked to levels of neurochemicals.
E. Antidepressant medications increase serotonin levels.

Correct choices are (A, B, D, and E). To promote client understanding and compliance, the teaching plan should include explanations about the origin and treatment options of OCD symptomology. Compulsions are behaviors that help relieve anxiety (A), which is a vague feeling related to unknown fears, that motivate behavior (B) to help the client cope and feel secure. All obsessions (C) do not result in compulsive behavior. OCD is supported by the neurophysiology theory, which attributes a diminished level of neurochemicals (D), particularly serotonin, and responds to selective serotonin reuptake inhibitors (SSRI).

A homeless person who is in the manic phase of bipolar disorder is admitted to the mental health unit. Which laboratory finding obtained on admission is most important for the nurse to report to the healthcare provider?
A. Decreased thyroid stimulating hormone level.
B. Elevated liver function profile.
C. Increased white blood cell count.
D. Decreased hematocrit and hemoglobin levels.
A. Decreased thyroid stimulating hormone level.

Hyperthyroidism causes an increased level of serum thyroid hormones (T3 and T4), which inhibit the release of TSH (A), so the client’s manic behavior may be related to an endocrine disorder. (B, C, and D) are abnormal findings that are commonly found in the homeless population because of poor sanitation, poor nutrition, and the prevalence of substance abuse.

A 30-year-old sales manager tells the nurse, “I am thinking about a job change. I don’t feel like I am living up to my potential.” Which of Maslow’s developmental stages is the sales manager attempting to achieve?
A. Self-Actualization.
B. Loving and Belonging.
C. Basic Needs.
D. Safety and Security.
A. Self-Actualization.

Self-actualization is the highest level of Maslow’s development stages, which is an attempt to fulfill one’s full potential (C). (B) is identifying support systems. (C) is the first level of Maslow’s developmental stages and is the foundation upon which higher needs rest. Individuals who feel safe and secure (D) in their environment perceive themselves as having physical safety and lack fear of harm.

The nurse is assessing a client’s intelligence. Which factor should the nurse remember during this part of the mental status exam?
A. Acute psychiatric illnesses impair intelligence.
B. Intelligence is influenced by social and cultural beliefs.
C. Poor concentration skills suggest limited intelligence.
D. The inability to think abstractly indicates limited intelligence.
B. Intelligence is influenced by social and cultural beliefs.

Social and cultural beliefs (B) have significant impact on intelligence. Chronic psychiatric illness may impair intelligence (A), especially if it remains untreated. Limited concentration does not suggest limited intelligence (C). Difficulties with abstractions are suggestive of psychotic thinking (D), not limited intelligence.

A male client with schizophrenia tells the nurse that the voices he hears are saying, “You must kill yourself.” To assist the client in coping with these thoughts, which response is best for the nurse to provide?
A. “Tell yourself that the voices are unreasonable.”
B. “Exercise when you hear the voices.”
C. “Talk to someone when you hear the voices.”
D. “The voices aren’t real, so ignore them.”
A. “Tell yourself that the voices are unreasonable.”

The nurse should teach the client to use self-talk to disprove the voices (A). Although (B) may be helpful, the client’s concrete thinking may make it difficult to understand this suggestion. Clients with schizophrenia have difficulty initiating interaction with others (C). Auditory hallucinations are often relentless, so it is difficult to ignore them (D).

A male adolescent is admitted with bipolar disorder after being released from jail for assault with a deadly weapon. When the nurse asks the teen to identify his reason for the assault, he replies, “Because he made me mad!” Which goal is best for the nurse to include in the client’s plan of care? The client will
A. outline methods for managing anger.
B. control impulsive actions toward self and others.
C. verbalize feelings when anger occurs.
D. recognize consequences for behaviors exhibited.
B. control impulsive actions toward self and others.

Those with bipolar disorder often exhibit poor impulse control, and the most important goal for this client at this time is to learn to control impulsive behavior (B) so that he can avert the social consequences related to such behaviors. (A, C, and D) are important goals, but they do not address the acute issue of impulse control, which is necessary to reduce the likelihood of harming self or others.

An adult male client who was admitted to the mental health unit yesterday tells the nurse that microchips were planted in his head for military surveillance of his every move. Which response is best for the nurse to provide?
A. “You are in the hospital, and I am the nurse caring for you.”
B. “It must be difficult for you to control your anxious feelings.”
C. “Go to occupational therapy and start a project.”
D. “You are not in a war area now; this is the United States.”
C. “Go to occupational therapy and start a project.”

Delusions often generate fear and isolation, so the nurse should help the client participate in activities that avoid focusing on the false belief and encourage interaction with others (C). Delusions are often well-fixed, and though (A) reinforces reality, it is argumentative and dismisses the client’s fears. It is often difficult for the client to recognize the relationship between delusions and anxiety (B), and the nurse should reassure the client that he is in a safe place. Dismissing delusional thinking (D) is unrealistic because neurochemical imbalances that cause positive symptoms of schizophrenia require antipsychotic drug therapy.

A client who is on a 30-day commitment to a drug rehabilitation unit, asks the nurse if he can go for a walk on the grounds of the treatment center. When he is told that his privileges do not include walking on the grounds, the client becomes verbally abusive. What responses are appropriate for the nurse to use? (Select all that apply.)
A. Call a staff member to escort the client to his room.
B. Ask the client to talk about what is causing him to be upset.
C. Ignore the client’s inappropriate behavior.
D. Remind the client of the unit rules.
E. Tell the client to talk to his healthcare provider about his privileges.
B. Ask the client to talk about what is causing him to be upset.
D. Remind the client of the unit rules.

Therapeutic responses to disruptive behavior should start with the nurse’s reflective interpretation of the client’s distress followed with an open-ended statement. “You seem upset; tell me about it.” A discussion of unit rules also provides an opportunity to explain how to obtain privileges to walk the grounds. The other options are not appropriate ways to respond to this situation.

Leave a Comment

Scroll to Top